Answer:
the answer is A 3 to 2
Step-by-step explanation:
but if it's wrong I'm sorry but if it's right I'm happy for you
Can someone pls help me
Answer:
10
Step-by-step explanation:
Answer: its an acute angle so its 90
Step-by-step explanation:
CAN SOMEONE HELPPPPP?!?!?!?!
Answer:
12
Step-by-step explanation:
6 ÷ 1/2 = 6 × 2 = 12
find the following for the given equation. r(t) = e−t, 8t2, 3 tan(t)
We can find several things for the given equation:
1. Magnitude of r(t):
| r(t) | = sqrt[ e^(-t)^2 + (8t^2)^2 + (3tan(t))^2 ]
2. Unit tangent vector T(t):
T(t) = r'(t) / | r'(t) |
where r'(t) = -e^(-t), 16t, 3sec^2(t)
3. Unit normal vector N(t):
N(t) = T'(t) / | T'(t) |
where T'(t) = -e^(-t), 16, 6sec^2(t)tan(t)
4. Unit binormal vector B(t):
B(t) = T(t) x N(t)
Note: "sec" stands for secant and "tan" stands for tangent.
To know more about Magnitude refer here
https://brainly.com/question/14452091#
#SPJ11
Noah said we cannot use long division to calculate 10 / 3 because there will always be a remainder do or disagree or agree why or why not
Answer:
disagree, its easy math, 10/3 =3.3 continued
-20-20+15
Help pleaseeee
Answer:
-25
Step-by-step explanation:
hope this helps
Answer:
-25
Step-by-step explanation:
-20-20 is -40.
-40+15 is -25
Hope this helps
Identify the angles of rotational symmetry for the figure.
Answer:
60°
Step-by-step explanation:
The internal angLe is 60°
If crytal room meaure 10. 5 ft long and 7 feet wide and 9 feet tall how much paint will be needed to paint her bedroom
Assuming that the walls of the room are 8 feet tall, and that you need one gallon of paint for every 400 square feet of wall space, then you would need 5.125 gallons of paint to paint the walls of the room.
1. Calculate the square footage of the walls:
10.5ft x 7ft x 2 (sides of walls) = 147 sq. ft
2. Divide the total wall square footage by the coverage area of the paint (400 sq. ft.):
147 sq. ft. ÷ 400 sq. ft. = 0.36 gallons
3. Add 10% extra paint for wastage:
0.36 gallons x 1.1 = 0.396 gallons
4. Round up to the nearest gallon:
0.396 gallons = 0.4 gallons
5. Multiply the gallon of paint by 4 (4 walls):
0.4 gallons x 4 = 1.6 gallons
6. Round up to the nearest gallon:
1.6 gallons = 2 gallons
Hence, you would need 5.125 gallons of paint to paint the walls of the room.
learn more about gallon here
https://brainly.com/question/19638640
#SPJ4
Find the derivative of the function f(x,y)=x² − 6xy+y² at the point (4, 3) in the direction in which the function increases in value most rapidly.
The gradient vector (−10, −18) represents the direction in which the function increases most rapidly at the point (4, 3).
To find the derivative of the function f(x,y)=x2−6xy+y2f(x,y)=x2−6xy+y2 with respect to xx and yy, we can take the partial derivatives ∂f∂x∂x∂f and ∂f∂y∂y∂f.
∂f∂x=2x−6y∂x∂f=2x−6y
∂f∂y=−6x+2y∂y∂f=−6x+2y
To determine the direction in which the function increases most rapidly at the point (4, 3), we need to find the gradient vector at that point. The gradient vector is given by the partial derivatives evaluated at the point (4, 3).
∇f(4,3)=(∂f∂x(4,3),∂f∂y(4,3))=(2(4)−6(3),−6(4)+2(3))=(−10,−18)∇f(4,3)=(∂x∂f(4,3),∂y∂f(4,3))=(2(4)−6(3),−6(4)+2(3))=(−10,−18)
Therefore, the derivative of the function at the point (4, 3) in the direction of maximum increase is (−10,−18)(−10,−18).
learn more about "vector ":- https://brainly.com/question/3184914
#SPJ11
Give the sin, cos, and tan for the point on the Unit Circle at 5pi/3°
Since you want to solve this using a calculator you have to options, if you have a advance calculator you can just directly input the nnumber and then it will give you the answer.
While using a standard calculator, we must first convert the radian measurement into degree measurement to find the sin, cos and tan of an angle. Let's start;
First, we must conver 5π/3 into degree measurement to do that we just have to multiply our converting factor 180/π (since 180° is equal to π)
\(\frac{5\pi}{3}(\frac{180}{\pi})=\frac{900\pi}{3\pi}=300\)Therefore 5π/3 is just equal to 300°.
Then using a CALCULATOR you can directly input the following;
\(\begin{gathered} \sin 300^{\circ}=-\frac{\sqrt[]{3}}{2}=-0.866 \\ \cos 300^{\circ}=\frac{1}{2}=0.5 \\ \tan 300^{\circ}=-\sqrt[]{3}=-1.732 \end{gathered}\)What is the equation of the line ? and how do i know ?
Answer:
The equation of the line is y=-16x+7
Step-by-step explanation:
I am not totally sure if that is what you meant by the equation of the line but that is my answer. I can change it if you are more specific.
Anyways, you need to find the slope because you already know the y-intercept, 7.
Slope=rise/run between two pretty points( A point that lines up on the grid)
So you must go from the bottom point and count how ever many are separating the two points. It turned out to be 16 and since the line is going down it is a negative. You then plug the numbers into the slope intercept form and that is your answer.
the difference between 75% of a number and 20% of the same number is 378.9 what is 40% of that number?
Thus, the 40% of the number is
\(688.909\cdot0.4=275.563\)find the pattern 7,10,14,19
Answer:
the answer is 25
Step-by-step explanation:
Students are given 3 minutes to complete each multiple-choice question on a test and 8 minutes for each free-
response question. There are 15 questions on the test and the students have been given 55 minutes to complete it.
Test Time
Number of Time per Item
Questions (minutes)
m
3
15-m
8
15
Multiple Choice
Free Response
Total
Which value could replace x in the table?
Total Time
(minutes)
3m
х
55
7-m
23-m
8(15 - m)
O 8(15) -
Answer:
See below :)
Step-by-step explanation:
The solution is, 12 multiple-choice questions are on the test.
What is simplification?Simplify simply means to make it simple. In mathematics, simply or simplification is reducing the expression/fraction/problem in a simpler form. It makes the problem easy with calculations and solving.
here, we have,
The right answer for the question that is being asked and shown above is that:
m + f = 15 ---> 3m + 3f = 45
3m + 5f = 51
now, we have.
3m + 3f = 45
3m + 5f = 51
-------------------
-2f = -6
f = 3
so, we get,
m + f = 15
m = 15 - 3
m = 12
So the correct answer is 12.
Hence, The solution is, 12 multiple-choice questions are on the test.
To learn more on simplification click:
brainly.com/question/28996879
#SPJ5
Complete question:
Students are given 3 minutes for each multiple-choice question and 5 minutes for each free-response question on a test. There are 15 questions on the test, and students are given 51 minutes to take it.
The system of equations shown can be used to find the number of multiple-choice questions, m, and the number of free-response questions, f, on the test.
m + f = 15
3m + 5f = 51
How many multiple-choice questions are on the test?
3
5
12
14
9 times the sum of 9b and 2
Answer:
9(9b+2)
Step-by-step explanation:
The words 'the sum of mean parenthesis.
And so 9 would be outside the parenthesis, then 9b and 2 would be inside.
35. Patrice cuts a piece of board 5 feet long from a board that is x feet long. Find an
expression that describes the length in feet of the original board after the cut.
a. 5 + x
c. 5-X
b. 5x
d. x-5
Help Quickly it’s in my packet
Answer:
i belive is D
Step-by-step explanation:
Which expression could be converted into a terminating decimal number?
Answer:
D
Step-by-step explanation:
\(\dfrac{\sqrt{4}}{\sqrt{25}}= \\\\\\\dfrac{2}{5}= \\\\\\0.4\)
Hope this helps!
give the dregree for the polynomial
Answer:
In the case of a polynomial with more than one variable, the degree is found by looking at each monomial within the polynomial, adding together all the exponents within a monomial, and choosing the largest sum of exponents.
Step-by-step explanation:
That sum is the degree of the polynomial.
the ratio 3:7 has 10 plant how much plants does 3:8 have
The number of plants 3 : 8 have if 3:7 has 10 plants is 11 plants.
How much plants does 3:8 have?Ratio refers to a number representing a comparison between two named things. It is also the relative magnitudes of two quantities usually expressed as a quotient.
So,
3 : 7
= 3 + 7
= 10 plants
Then,
3 : 8
= 3 + 8
= 11 plants
In conclusion, 3 ratio 8 has 11 plants.
Read more on ratio:
https://brainly.com/question/2328454
#SPJ1
help meeeeeeeeeeeeeeeeeee pleaseeeeeeeeeeeeeee!!!!!!!!!!!!!!help meeeeeeeeeeeeeeeeeee pleaseeeeeeeeeeeeeee!!!!!!!!!!!!!!help meeeeeeeeeeeeeeeeeee pleaseeeeeeeeeeeeeee!!!!!!!!!!!!!!help meeeeeeeeeeeeeeeeeee pleaseeeeeeeeeeeeeee!!!!!!!!!!!!!!
Answer:
129723
Explanation:
\(1800(1+0.13)^{35} \approx 129723\)
What is the domain of the function y - 2e graphed below?
5
2
5 4 -3 -2 -11
1
2
3
4
Ui
X
-2
all real numbers greater than 2
O all positive real numbers
O all negative real numbers
all real numbers
The domain of the function is all real numbers.
Option 4 is correct
What is Domain?The range of values that we are permitted to enter into our function is known as the domain of a function. The x values for a function like f make up this set (x).
A function's range is the collection of values it can take as input. After we enter an x value, the function outputs this sequence of values. The y values are those. A function can be compared to a piece of equipment on an assembly line. A few screws and nuts are located on one end of the assembly line, while a car is located on the other.
As per the given data:
The function is \(y = 2e^x\)
This function is defined for all the values of x as y never becomes undefined.
Hence, the domain of the given function is (-∞, ∞)
To learn more on domain, click:
brainly.com/question/28135761
#SPJ7
Evaluate the integral by making an appropriate change of variables.
∫∫R5(x+y)ex2−y2dA, where R is the rectangle enclosed by the lines x−y=0,x−y=3,x+y=0, and x+y=4.
The integral ∫∫R5(x+y)ex2−y2dA, where R is the rectangle enclosed by the lines x−y=0,x−y=3,x+y=0, and x+y=4, can be evaluated by making the change of variables u = x + y and v = x - y, which gives us the Jacobian of the transformation as |J| = 1/2.
To evaluate the integral using the change of variables, we first need to find the bounds of integration for the new variables u and v. Using the equations of the lines that bound the rectangle R, we can rewrite them in terms of u and v as v = ±(3 - u) and v = ±u. These equations represent the four lines that form the new rectangle R' in the uv-plane.
The integral can now be rewritten as ∫∫R'5(u/2)eu2/2dvdu. The limits of integration for v are from -(3 - u) to (3 - u) for the bottom and top sides of the rectangle R', and from -u to u for the left and right sides. The limits of integration for u are from 0 to 4.
After integrating with respect to v, we get ∫(3-u)^u(-3+u)5(u/2)eu2/2dvdu + ∫u^-u^3 5(u/2)eu2/2dvdu. These integrals can be solved by using the substitution method. Finally, we get the answer as 17(e^16 - e^(4/3))/12.
learn more about "Integral":-https://brainly.com/question/30094386
#SPJ11
the mayor of a city randomly chooses 4 of the 20 wards in her city and surveys all the voters in those selected wards.
The sample space is 4845.Therefore, option C is correct.
Given: The mayor of a city randomly chooses 4 of the 20 wards in her city and surveys all the voters in those selected wards.
Solution: We have a sample size of 4 wards out of a total of 20 wards. The mayor has chosen all the voters in those 4 wards. So, the sampling method used here is stratified sampling as the mayor chose the voters of specific 4 wards.
Sample space = Total number of ways to select 4 wards out of 20 wards= C(20,4) = 20!/(4!(20 - 4)!) = 4845
Hence, the sample space is 4845.Therefore, option C is correct.
Learn more about sample space visit:
brainly.com/question/30206035
#SPJ11
find the value of each of the lettered angles in the diagram below
Answer:
Angle a = 114 Angle b = 114 Angle c = 66 Angle d = 66
Step-by-step explanation:
180 - 48 = 132
132/2= 66
180 - 66 = 114
A pair of vertical angles has measures (2y 5)° and (4y)°. What is the value of y? −52 −25 25 52.
Vertical angles are a pair of non-adjacent angles formed by the intersection of two lines. They are equal in measure. The value of y is 25.
In this case, we are given two vertical angles with measures (2y + 5)° and (4y)°. Since they are equal, we can set up an equation to find the value of y.
(2y + 5)° = (4y)°
To solve for y, we can start by subtracting (2y)° from both sides of the equation:
5° = 2y°
Next, divide both sides of the equation by 2 to isolate y:
2.5° = y°
Therefore, the value of y is 2.5° or 25.
Learn more about angles here:
https://brainly.com/question/31818999
#SPJ11
write the repeating decimal as a fraction 0.75
the 75 is repeating
Let x = 0.757575... . Then
100x = 75.757575...
and so
100x - x = 75.757575... - 0.757575...
99x = 75
x = 75/99 = 25/33
what difficulty did you encounter in determining polynomials?
Answer:
I didn’t really encounter any difficulty in determining a polynomial. The issue was that numbers with negative exponents or those which had square roots in them aren’t referred to as polynomials.
The only minor issue was solving problems with numbers which had different exponents in them and needed some form of simplification.
In solving the beam equation, you determined that the general solution is X. y = = 1/2x^4 - 1/6q₁ x^3 + 1/2 x. Given that y'' (1) = 3 determine q1 ₁
We have the general solution of the beam equation: y = 1/2 x⁴ - (1/6)q₁ x³ + (1/2) x
Given that y'' (1) = 3
So we can find the second derivative of y: y' = 2x³ - (1/2)q₁x² + (1/2)and y'' = 6x² - q₁x
Therefore, y''(1) = 6 - q₁
From the given information: y''(1) = 3
Putting this value into the above equation:3 = 6 - q₁=> q₁ = 6 - 3=> q₁ = 3
The value of q₁ is 3.
To know more about derivative visit :
https://brainly.com/question/30466081
#SPJ11
13/21 as a decimal rounded to 3 decimal places
Answer:
\({ \tt{ \frac{13}{21} \dashrightarrow \: { \boxed{0.619}}}}\)
13/21 as a decimal rounded to 3 decimal places is 0.619.
What is decimal?Decimals are numbers that have two components, a whole number component and a fractional component, which are separated by a decimal point.
Given:
A fraction: 13/21.
Simplifying,
13/21,
= 0.619047619
= 0.619 up to 3 decimal places.
Therefore, 0.619 up to 3 decimal places.
To learn more about the decimal;
brainly.com/question/109147
#SPJ2
a person stands on a scale in an elevator. as the elevator starts, the scale has a constant reading of 590 n. as the elevator later stops, the scale reading is 386 n. assume the magnitude of the acceleration is the same during starting and stopping.
Assuming the weight and magnitude of the person, we can conclude that the weight of the person is 491N.
What is the magnitude?The magnitude or size of a mathematical object determines whether it is larger or smaller than other objects of the same kind in mathematics.
Formally speaking, the magnitude of an object is the visible outcome of an ordering—of the class of objects to which it belongs.
So, the weight of the person will be:
The scale measures the passenger's normal upward force as it is applied by the floor.
When moving upward, the most force is generated (when starting an upward trip or ending a downward trip).
When there is a downhill acceleration, the normal force is at its lowest.
For any such circumstance:
∑F y =ma y becomes for starting +591N−mg=+ma and for stopping +391N−mg=−ma.
Where a stands for the acceleration's magnitude.
To get rid of a and find mg, add these two simultaneous equations:
+591N−mg+391N−mg=0
OR
982N–2mg=0
Fg = mg = 982N/2 = 491N
Therefore, assuming the weight and magnitude of the person, we can conclude that the weight of the person is 491N.
Know more about magnitude here:
https://brainly.com/question/24468862
#SPJ4
Complete question:
A person stands on a scale in an elevator. As the elevator starts, the scale has a constant reading of 591N. As the elevator later stops, the scale reading is 391N. Assuming the magnitude of the acceleration is the same during starting and stopping, determine the weight of the person.
The nile river is 6690 kilometers long.This is 394 kilometers longer than the Amazon River.How long is the Amazon River?
Answer:
The nile river is 6690 kilometers long.This is 394 kilometers longer than the Amazon River.How long is the Amazon River?
6690-394=6296 kilometers
Step-by-step explanation:
Answer:
The Amazon river is 6296 kilometers long.